Difference between revisions of "2023 AMC 10A Problems/Problem 19"

(added problem)
Line 2: Line 2:
  
 
<math>\text{A) } \frac{1}{4} \qquad \text{B) } \frac{1}{2} \qquad \text{C) } \frac{3}{4}  \qquad \text{D) } \frac{2}{3} \qquad  \text{E) } 1</math>
 
<math>\text{A) } \frac{1}{4} \qquad \text{B) } \frac{1}{2} \qquad \text{C) } \frac{3}{4}  \qquad \text{D) } \frac{2}{3} \qquad  \text{E) } 1</math>
 +
 +
 +
== Video Solution 1 by OmegaLearn ==
 +
https://youtu.be/88F18qth0xI

Revision as of 16:15, 9 November 2023

The line segment formed by $A(1, 2)$ and $B(3, 3)$ is rotated to the line segment formed by $A'(3, 1)$ and $B'(4, 3)$ about the point $P(r, s)$. What is $|r-s|$?

$\text{A) } \frac{1}{4} \qquad \text{B) } \frac{1}{2} \qquad \text{C) } \frac{3}{4}   \qquad \text{D) } \frac{2}{3} \qquad   \text{E) } 1$


Video Solution 1 by OmegaLearn

https://youtu.be/88F18qth0xI